Đến nội dung

toanND

toanND

Đăng ký: 15-12-2018
Offline Đăng nhập: 12-09-2021 - 22:26
-----

#728967 IJ đi qua điểm chính giữa cung BC

Gửi bởi toanND trong 17-07-2021 - 19:08

Lời giải của mình: 

Gọi $E$ là giao điểm của $AC$ $BD$. $F$ là giao điểm của $AB$ $CD$.

Gọi $L$ là điểm chính giữa cung $BC$ không chứa $A,D$, $LI$ cắt $(O)$ lần thứ hai tại $T$.

Gọi $X,Y$ lần lượt là tâm các đường tròn bàng tiếp góc $C,B$ của $\triangle ABC, \triangle BDC$

Nếu gọi $S$ là tâm đường tròn nội tiếp tam giác $ABC$ thì ta có $\angle XBS = \angle XAS = 90^0 \Rightarrow XBSA$ nội tiếp

$\Rightarrow \angle BXI = \angle BAS = \angle BTI \Rightarrow$ Tứ giác $XBIT$ nội tiếp.

Tương tự ta cũng chứng minh được tứ giác $TICY$ nội tiếp.

Từ đó ta có $\angle BXC = \angle BTL = \angle CTL = \angle BYC \Rightarrow XBCY$ nội tiếp.

Do $XB, BJ$ đều là phân giác ngoài góc $ABC$ nên $X,B,J$ thẳng hàng. Tương tự, $Y,C,J$ thẳng hàng.

Theo tính chất trục đẳng phương thì trục đẳng phương của ba đường tròn xanh phải đồng quy, tức là $XB, TI, YC$ đồng quy. Mà $XB$ cắt $YC$ tại $J$, suy ra $T,I,J$ thẳng thàng.

Do đó $I,J,L$ thẳng hàng hay ta có $đpcm \blacksquare$ 

comeback 1.PNG

P/S: Nếu ta dựng một đường tròn $\omega$ tiếp xúc với $AE, DE$ và tiếp xúc trong với $(O)$ thì $T$ là tiếp điểm của $\omega$ với $(O)$

comback 2.PNG




#724550 CMR $ TM // BC $

Gửi bởi toanND trong 08-08-2019 - 09:38

Bài này có thể giải bằng định lý Pascal




#724490 $\widehat{NLP}=90^0$

Gửi bởi toanND trong 05-08-2019 - 21:44

capture NVL.PNG

Gọi AQ là đường đối trung ứng với đỉnh A của tam giác ABC. AQ cắt (O) lần thứ hai tại T

$\Rightarrow$ Tứ giác ABTC điều hòa $\Rightarrow$ SA tiếp xúc với (O) tại A (S là giao của OL với BC).

Gọi $H_{a}$ là chân đường cao kẻ từ A của tam giác ABC. K là giao của OL với $AH_{a}$.

$\Rightarrow$ K là trực tâm tam giác AQS $\Rightarrow QK \perp AS$

Gọi M, I lần lượt là trung điểm của BC, $H_{b}H_{c}$. Khi đó M, I, N thẳng hàng và $MN\perp H_{b}H_{c}$

Mặt khác dễ thấy $AS \parallel H_bH_c$ $\Rightarrow QK \parallel MN$

$KLQH_a$ nội tiếp $\Rightarrow \widehat{H_aLQ}=\widehat{H_aKQ}=\widehat{H_aNI}(KQ\parallel MN)$

$\Rightarrow NILH_a$ nội tiếp. Mặt khác $I,H_a$ cùng nằm trên đường tròn đường kính NP

Suy ra L nằm trên đường tròn đường kính NP$\Rightarrow \widehat{NLP}=90^0$ (đpcm)  :like




#724419 $1+\sqrt{\frac{2+1}{2}}+\sq...

Gửi bởi toanND trong 01-08-2019 - 21:05

 chỗ $\left ( 1+\frac{1}{n^{2}} \right )^{n}> 1+n.\frac{1}{n^{2}}$ này là sao bạn

Bất đẳng thức Bernoulli đó




#723712 Bất đẳng thức

Gửi bởi toanND trong 13-07-2019 - 09:53

  • Trước hết, ta chứng minh bđt phụ sau:

Cho các số thực dương a,b,c. Chứng minh rằng $\frac{a}{c}+\frac{c}{b}+\frac{b}{a}\geq\frac{\sqrt{3(a^2+b^2+c^2)}}{\sqrt[3]{abc}}$ (1)

Thật vậy $(1)\Leftrightarrow \frac{a^2}{c^2}+\frac{c^2}{b^2}+\frac{b^2}{a^2}+2(\frac{a}{b}+\frac{c}{a}+\frac{b}{c})\geq\frac{3(a^2+b^2+c^2)}{\sqrt[3]{(abc)^2}}$

Áp dụng BĐT AM-GM ta có $\frac{a^2}{c^2}+\frac{a}{b}+\frac{a}{b}\geq3\sqrt[3]{\frac{a^4}{b^2c^2}}=\frac{3a^2}{\sqrt[3]{(abc)^2}}$

Tương tự với các bđt còn lại, sau đó cộng lại ta có đpcm.

  • Trở lại bài toán, áp dụng bđt phụ trên thì ta cần chứng minh $\frac{\sqrt{3(a^2+b^2+c^2)}}{\sqrt[3]{abc}}+\sqrt[3]{abc}\geq\frac{10}{9(a^2+b^2+c^2)}$

Áp dụng bđt AM-GM ta có

$\frac{\sqrt{3(a^2+b^2+c^2)}}{\sqrt[3]{abc}}+\sqrt[3]{abc}=\frac{\sqrt{3(a^2+b^2+c^2)}}{\sqrt[3]{abc}}+9\sqrt[3]{abc}-8\sqrt[3]{abc}\geq2\sqrt{9\sqrt{3(a^2+b^2+c^2)}}-\frac{8}{3}(a+b+c)=6\sqrt[4]{3(a^2+b^2+c^2)}-\frac{8}{3}$

Đặt $t=\sqrt[4]{3(a^2+b^2+c^2)}\geq\sqrt[4]{(a+b+c)^2}=1$

Ta chỉ cần chứng minh $6t-\frac{8}{3}\geq\frac{10}{3t^4}\Leftrightarrow t^4(9t-4)\geq5$ (đúng do $t\geq1$ )

Vậy ta có đpcm. ~O)

 




#723698 Một số bài toàn bất đẳng thức mình cần được giúp đỡ!

Gửi bởi toanND trong 12-07-2019 - 16:52

$\boxed{16}$

Áp dụng BĐT Cauchy Schwarz ta có

$\frac{a}{4b^2+1}+\frac{b}{4c^2+1}+\frac{c}{4a^2+1}=\frac{a^3}{4a^2b^2+a^2}+\frac{b^3}{4b^2c^2+c^2}+\frac{c^3}{4c^2a^2+c^2}\geq \frac{(a\sqrt{a}+b\sqrt{b}+c\sqrt{c})^2}{4(a^2b^2+b^2c^2+c^2a^2)+a^2+b^2+c^2}$

Ta chỉ cần chứng minh 

$4(a^2b^2+b^2c^2+c^2a^2)+a^2+b^2+c^2\leq 1 =(a+b+c)^2$

$\Leftrightarrow ab(1-2ab)+bc(1-2bc)+ca(1-2ca)\geq0$

Lại có $ab>0$ , $ab\leq\frac{(a+b)^2}{4}<\frac{(a+b+c)^2}{4}$ $\Rightarrow ab<\frac{1}{4}\Rightarrow ab(1-2ab)>0$

Tương tự ta có đpcm

Dấu = không xảy ra




#723607 CM : $\frac{x^2y}{y} + \frac{y^2z...

Gửi bởi toanND trong 08-07-2019 - 21:32

Cho 0 < x ≤ y ≤ z. CM : $\frac{x^2y}{y} + \frac{y^2z}{x} + \frac{z^2x}{y} \geqslant x^2 + y^2 + z^2$

Hình như điều kiện phải là $x\geq y\geq z>0$. Đây là VMO 1991




#723438 Cho các số thực dương a, b, c

Gửi bởi toanND trong 02-07-2019 - 09:14

Cho các số thực dương a, b, c. Chứng minh rằng 

$\frac{1}{(a+b)^2}+\frac{1}{(b+c)^2}+\frac{1}{(c+a)^2}\geq \frac{3\sqrt{3abc(a+b+c)}(a+b+c)^2}{4(ab+bc+ca)^3}$




#723404 Hê thức lượng

Gửi bởi toanND trong 29-06-2019 - 22:28

$\boxed{3}$

Kẻ EK, FI lần lượt vuông góc với AB, AC tại K, I.

Ta có $\frac{AK}{BK}=\frac{AK.AB}{BK.AB}=\frac{EA^2}{EB^2}=4=\frac{HC}{HB}$

theo định lý Thales đảo $\Rightarrow HK||AC$. Tương tự $HI\parallel AB$

$\Rightarrow AIHK$ là hình bình hành

Ta có $\bigtriangleup AEB\sim \bigtriangleup CFA \Rightarrow \measuredangle BAE=\measuredangle ACF=\measuredangle AFI\Rightarrow \bigtriangleup AEK\sim \bigtriangleup FAI$

$\Rightarrow \frac{EK}{AK}=\frac{AI}{FI}\Rightarrow \frac{EK}{AI}=\frac{AK}{FI}$. Mà $AK = HI,AI=HK$

$\Rightarrow \frac{EK}{HK}=\frac{HI}{IF}$. Lại có $\measuredangle EKH=\measuredangle BKH +90^0=\measuredangle HIC+90^0=\measuredangle HIF$

Do đó $\bigtriangleup EKH\sim\bigtriangleup HIF\Rightarrow\measuredangle HEK=\measuredangle IHF$

Vậy $\measuredangle EHF=\measuredangle EHK+\measuredangle KHI+\measuredangle IHF=(\measuredangle EHK+\measuredangle HEK)+\measuredangle BKH=180^0-\measuredangle EKH+\measuredangle BKH=180^0-90^0=90^0$

 




#723400 Hê thức lượng

Gửi bởi toanND trong 29-06-2019 - 17:40

$\boxed{\text{1}}$

hethuc1.PNG

a. Dễ thấy tứ giác KIHB nội tiếp $\rightarrow CI.CK=CH.CB=CA^2$

b. theo tính chất đối xứng thì $\measuredangle BDC=\measuredangle BAC=90^0;AC=DC$

Vì $\measuredangle BKC=\measuredangle BAC=\measuredangle BDC=90^0$ nên $B,K,A,C,D$ cùng thuộc một đường tròn

$\Rightarrow \measuredangle CKD=\measuredangle DAC=\measuredangle ADC=\measuredangle AKC$

ta có đpcm

 




#723109 bất đẳng thức

Gửi bởi toanND trong 17-06-2019 - 12:51

Áp dụng BĐT $b^2+bc+c^2\geq\frac{3}{4}(b+c)^2$

Ta có $15\geq3a^2+4(b^2+bc+c^2)\geq3[a^2+(b+c)^2]\geq\frac{3}{2}(a+b+c)^2$

$\Rightarrow (a+b+c)^2\leq10\Rightarrow-\sqrt{10}\leq a+b+c\leq \sqrt{10}$

Từ đó ta có min , max

e tự tìm dấu = nhé




#723098 bất đẳng thức

Gửi bởi toanND trong 16-06-2019 - 22:23

BÀI 3. Áp dụng BĐT Bunhiacopxki ta có $(a^2+b)(1+\frac{1}{b})\geq(a+1)^2$

Tương tự với các BĐT còn lại rồi nhân lại ta có $(a^2+b)(b^2+c)(c^2+a)\frac{(a+1)(b+1)(c+1)}{abc}\geq(a+1)^2(b+1)^2(c+1)^2$

$\Leftrightarrow (a^2+b)(b^2+c)(c^2+a)\geq abc(a+1)(b+1)(c+1)$

Dấu = xảy ra khi a =b =c 




#723048 bất đẳng thức

Gửi bởi toanND trong 14-06-2019 - 16:37

Ngưỡng mộ anh toanND quá, anh chỉ cho em cách học giỏi bđt với ạ

Kiếm sách với tài liệu mà đọc thôi e :ukliam2:




#723037 bất đẳng thức

Gửi bởi toanND trong 14-06-2019 - 13:22

Em cảm ơn anh toannd, bài 1 câu a anh có thể giải bằng cauchy dc k ah

Có thể dùng AM - GM (Cauchy) kiểu này : $\frac{1}{a}+\frac{1}{b}\geq\frac{4}{a+b}; \frac{1}{ab}\geq\frac{4}{(a+b)^2}$

Áp dụng hai BĐT trên, ta biến đổi biểu thức P như sau:

$P=(1+\frac{1}{a}+\frac{1}{b}+\frac{1}{ab})(1+\frac{1}{c}+\frac{1}{d}+\frac{1}{cd})\geq[1+\frac{4}{a+b}+\frac{4}{(a+b)^2}][1+\frac{4}{c+d}+\frac{4}{(c+d)^2}]=[(\frac{2}{a+b}+1)(\frac{2}{c+d}+1)] ^2 ]$

Đặt $A=[(\frac{2}{a+b}+1)(\frac{2}{c+d}+1)]^2$

$\Rightarrow A=[\frac{4}{(a+b)(c+d)}+\frac{2}{a+b}+\frac{2}{c+d}+1]^2\geq[\frac{16}{(a+b+c+d)^2}+\frac{8}{a+b+c+d}+1]^2=625$

$\Rightarrow P\geq A\geq625$  ~O)




#723026 bất đẳng thức

Gửi bởi toanND trong 14-06-2019 - 08:50

BÀI 1

a. Áp dụng BĐT Holder ta có: $P=(\frac{1}{a}+1)(\frac{1}{b}+1)(\frac{1}{c}+1)(\frac{1}{d}+1)\geq (\sqrt[4]{\frac{1}{abcd}}+1)^{4}$

Mặt khác theo BĐT AM-GM: $\sqrt[4]{abcd}\leq \frac{a+b+c+d}{4}=\frac{1}{4}$

$\Rightarrow P\geq(4+1)^{4}=625$

Vậy $minP=625$ khi $a=b=c=d= \frac{1}{4}$

b. Ta có $Q=\frac{a(b+c+d)}{\frac{a^{2}}{3}+b^{2}+\frac{a^{2}}{3}+c^{2}+\frac{a^{2}}{3}+d^{2}}\leq\frac{a(b+c+d)}{\frac{2}{\sqrt{3}}(ab+ac+ad)}=\frac{\sqrt{3}}{2}$

Vậy $maxQ = \frac{\sqrt{3}}{2}$ khi ..........

BÀI 2. Ý tưởng cũng giống bài 1b thôi e  :closedeyes: